Você está na página 1de 19

MATH 1530 ABSTRACT ALGEBRA

Selected solutions to problems

Problem Set 2

2. Define a relation ∼ on R given by a ∼ b if a − b ∈ Z.

(a) Prove that ∼ is an equivalence relation.


(b) Let R/Z denote the set of equivalence classes of ∼. Prove that the binary operation +
on R/Z given by
a+b=a+b
is well-defined.
(c) Is (R/Z, +) a group?

Solution. (I will omit the proof that R/Z is a group.) Given a, b, c ∈ R, note:

(a) a − a = 0 ∈ Z;
(b) a − b ∈ Z implies b − a = −(a − b) ∈ Z; and
(c) if a − b ∈ Z and b − c ∈ Z then a − c = (a − b) − (b − c) ∈ Z;

so ∼ is an equivalence relation.
Next, we show that the binary operation + on R/Z given by a + b = a + b is well defined.
Suppose a = a0 and b = b0 for some a, b, a0 , b0 ∈ R. We wish to show a + b = a0 + b0 .
The assumptions imply a − a0 ∈ Z and b − b0 ∈ Z, so

(a + b) − (a0 + b0 ) = (a − a0 ) + (b − b0 ) ∈ Z.

So
a + b = a0 + b0 ,
as desired.

1
Problem Set 3

2. Write out the full multiplication table for the group S3 .


It should be a 6 × 6 table whose entries are elements of S3 , and whose rows and columns are
indexed by the elements of S3 . Please write all elements using cycle notation.

Solution.
· id (12) (13) (23) (123) (132)
id id (12) (13) (23) (123) (132)
(12) (12) id (132) (231) (23) (13)
(13) (13) (123) id (231) (12) (23)
(23) (23) (132) (123) id (13) (12)
(123) (123) (13) (23) (12) (132) id
(132) (132) (23) (12) (13) id (123)

3. Is it true that for all n ≥ 1, every element of Sn has order at most n? Either prove it, or give
a counterexample.

Solution. No. We saw in the Youtube lecture that the order of an element σ ∈ Sn is the
least common multiple of all of the numbers appearing as lengths of some cycle in σ, when σ
is expressed in cycle notation.
Thus the order of (12)(345) ∈ S5 is 6.

5. Let (G, ∗) and (H, ◦) be groups, and let φ : G → H be an isomorphism. Let φ−1 : H → G
denote the inverse of φ. In other words, φ−1 (h) = g whenever φ(g) = h.
Prove that φ−1 is also an isomorphism.

Solution. Let us use the fact from class:


Fact. A map f : A → B is bijective if and only if there exists an inverse, i.e. a map
g : B → A with f ◦ g = idB and g ◦ f = idA , where idA : A → A and idB : B → B denote
the respective identity maps. This fact implies that the inverse of a bijective map is again
a bijection. So φ−1 is a bijection, and it remains to show φ−1 is a homomorphism of groups.
Indeed, given h, h0 ∈ H, let g, g 0 ∈ G be the unique elements of G such that φ(g) = h and
φ(g 0 ) = h0 ; such g, g 0 exist and are unique since φ is bijective. Then note φ(gg 0 ) = hh0 since
φ is a homomorphism.
Then
φ−1 (hh0 ) = gg 0 = φ−1 (h)φ−1 (h0 ),

2
as desired.

6. Prove that isomorphism is an equivalence relation on groups.

Solution. It’s reflexive since idG : G → G is an isomorphism from G to itself. The previous
problem shows that it is symmetric. For transitivity, it suffices to show that a composition
of isomorphisms is again an isomorphism. In other words, we show that if φ : G → H and
ψ : H → K are isomorphisms, then so is ψ ◦ φ : G → K.
The fact that ψ ◦ φ is a bijection follows from the fact that compositions of bijections are
bijections. I omit the proof. To check that ψ ◦ φ is a homomorphism: given g, g 0 ∈ G, note

(ψ ◦ φ)(gg 0 ) = ψ(φ(g)φ(g 0 )) = (ψ ◦ φ)(g)(ψ ◦ φ)(g 0 ),

where we used in turn the facts that ψ and φ are homomorphisms.

3
Problem Set 4

2. How many homomorphisms Z → Z/3Z are there? How many homomorphisms Z/3Z → Z
are there? Justify your answers briefly.

Solution. There are three homomorphisms Z → Z/3Z, since we showed in class that for
any group G and any g ∈ G, there is a unique homomorphism φ : Z → G such that φ(1) = g.
There is one homomorphism Z/3Z → Z. We mentioned in class that for any pair of groups
G and H, the map sending everything in G to 1H is always a homomorphism (check this), so
there is at least one such homomorphism.
On the other hand, suppose φ : Z/3Z → Z is any homomorphism; we’ll show that φ(n) = 0
for all n ∈ Z/3Z, so φ was in fact the one homomorphism we already identified.
It suffices to show that φ(1) = 0, since then φ(n) = nφ(1) = 0. We note

0 = φ(0) = φ(1 + 1 + 1) = φ(1) + φ(1) + φ(1).

So φ(1) = 0.

5. Does R have any subgroups isomorphic to Z2 ? Prove your answer.

Solution. Yes, let √


H = {a + b 2 : a, b ∈ Z}.
We claim H ≤ R is a subgroup isomorphic to Z2 . I will omit the proof that H is a subgroup.1
Let us now consider the map
φ : Z2 → H

given by (a, b) 7→ a + b 2. This map is a homomorphism: indeed

φ((a, b) + (c, d)) = φ((a + c, b + d)) = (a + c) + (b + d) 2 = φ((a, b)) + φ((c, d)).

It is surjective by construction. As for injectivity, suppose φ((a, b)) = φ((c, d)); we wish to
show (a, b) = (c, d). We have
√ √ √
a + b 2 = c + d 2 implies a − c = (d − b) 2.

The left hand side is an integer and the right hand side is irrational unless d − b = 0. We
conclude d − b = 0, and therefore a − c = 0, as desired. Therefore φ is an isomorphism.

1
In fact, it follows from the fact that H = im(φ) where φ is defined below; but we had not isolated and proved
the fact that im(φ) is always a subgroup when this problem was assigned.)

4
6. Does Q have any subgroups isomorphic to Z2 ? Prove your answer.

Solution. We claim the answer is no. Suppose on the contrary that φ : Z2 → H was an
isomorphism from Z2 to some subgroup H. Let us write the rational numbers φ(1, 0) and
φ(0, 1) with a common denominator, say

φ(1, 0) = a/n, φ(0, 1) = b/n

for some a, b, n ∈ Z with n 6= 0. Then

φ(b, 0) = φ(0, a) = ab/n.

Since φ is injective, we conclude (b, 0) = (0, a), so a = b = 0. But this contradicts that
φ(1, 0) 6= 0 and φ(0, 1) 6= 0 by injectivity of φ.

5
Problem Set 5

2. (a) Let G be a group acting on a set A. The stabilizer of an element a ∈ A, denoted Ga , is


defined to be the set
Ga = {g ∈ G : g · a = a}.
Prove that the stabilizer is a subgroup of G.
In class on Thursday February 23, you considered the action of the group G of rotations on
the power set P(R2 ) of R2 .

(b) Which elements of P(R2 ) have stabilizer equal to G? Justify your answer briefly.
(c) (optional, extra) Does there exist any S ∈ P(R2 ) with infinite cyclic stabilizer?

Solution. Note 1 ∈ Ga . Next, if g, h ∈ Ga , then we have

(gh−1 ) · a = g(h−1 · a) = g · a = a,

so gh−1 ∈ Ga . (In general, if h ∈ G and h · b = c for some b, c ∈ A, then applying h−1 on


both sides yields that b = h−1 · c. Then in particular h · a = a implies h−1 · a = a.) Thus Ga
is a subgroup by the subgroup criterion.
For part (b), suppose S ⊆ R2 has GS = G. If p ∈ S then every point obtained by rotating p
about the origin must also be in S; in other words, the entire locus of points

{x ∈ R2 : ||x|| = ||p||}

lies in S. Thus S is a union of such loci, i.e. it is a union of circles centered at the origin,
possibly together with the origin itself.

4. Let p be any prime number. Prove that every group of order p is isomorphic to Z/pZ.

Solution. Let G be a group of order p. Since p ≥ 2, G has some nonidentity element, say x.
Then hxi is a subgroup of G whose cardinality is greater than 1 and divides p (by Lagrange).
Therefore hxi = G, so G is cyclic, and every cyclic group of order p is isomorphic to Z/pZ,
as shown in class.

5. We showed in an earlier lecture that

Z(G) = {x ∈ G : gx = xg for all g ∈ G}

is a subgroup of G, called the center of G. Prove that Z(G) is normal, and that it consists
precisely of the elements of G whose conjugacy classes have size 1.

6
Solution. The first claim follows from the second, since we showed that if a subgroup is
a union of conjugacy classes then it is normal. For the second claim, note that an element
x ∈ G has conjugacy class {x} if and only if

gxg −1 = x for all g ∈ G,

equivalently, if gx = xg for all g ∈ G, equivalently if x ∈ Z(G), as desired.

6. List the conjugacy classes of the dihedral group D12 . Draw the lattice of subgroups of D12
and indicate in your lattice which subgroups are normal. (No proofs necessary.)

Solution. I will leave the full drawing of the lattice to you. The conjugacy classes are

{1}, {r, r5 }, {r2 , r4 }, {r3 }, {s, sr2 , sr4 }, {sr, sr3 , sr5 }.

(Having listed these conjugacy classes, you may wish to use this list to check which subgroups
are normal. Namely, you simply check whether your subgroup is made up of conjugacy
classes.)

7
Problem Set 6
1. An element g of a group G is called torsion if it has finite order, and G is called torsion-free
if its only torsion element is the identity.
Let A be an abelian group and let N be the set of its torsion elements. Prove that N is a
subgroup and that A/N is torsion-free.

Solution. 1 has order 1, so 1 ∈ N . Now given a, b ∈ N , let us show ab−1 ∈ N . Since a, b are
torsion, there exist integers m, n ≥ 1 with am = bn = 1. Then note
(ab−1 )mn = amn b−mn = (am )n (bn )−m = 1,
where the first equality follows from the fact that A is abelian. Thus the subgroup criterion
implies that N is a subgroup.
Next, we show A/N is torsion-free. Given aN ∈ A/N , suppose aN is torsion; we wish to show
aN = N is the identity. Let m ≥ 1 be such that (aN )m = N . Then am N = N , implying
am ∈ N .
In other words, am itself is torsion in A, so there exists an integer n ≥ 1 with (am )n = 1.
Therefore amn = 1 and so a is torsion itself. So a ∈ N , and aN = N as desired.

2. (No proofs necessary) Let N = Z(D12 ) E D12 .


(a) List the elements of N . You may use Dummit and Foote Problem 4 on p.28, on Home-
work 3, or combine your answers from Problems 5 and 6 from Homework 5.
Now list the elements of D12 /N ; there should be six of them.

Solution. N = {1, r3 }, and


D12 /N = {N, rN, r2 N, sN, srN, sr2 N }.

(b) Write out the multiplication table for the group D12 /N . This should be a 6 × 6 table,
all of whose entries are taken from your list of elements of D12 /N from part (a).

· N rN r2 N sN srN sr2 N
N N rN r2 N sN srN sr2 N
rN rN r2 N N sr2 N sN srN
Solution. r2 N r2 N N rN srN sr2 N sN
sN sN srN sr2 N N rN r2 N
srN srN sr2 N sN r2 N N rN
sr2 N sr2 N sN srN rN r2 N N

8
3. Number the three diagonals of a regular hexagon as shown. Let D12 act on the set {1, 2, 3}
via g · i = j if g ∈ D12 takes the diagonal i to diagonal j.
Let φ : D12 → S3 be the permutation representation of this action. Use φ to prove that

D12 /N ∼
= S3 .

Feel free to appeal to your geometric intuition. (In light of this fact, you may wish to compare
your multiplication table for D12 /N with your multiplication table for S3 from Homework 3.)

2 1
3

Solution. We know that φ is indeed a homomorphism, as stated in the guest lecture. It


suffices to check that φ is surjective, and that ker φ = N , for then the desired isomorphism
follows from the First Isomorphism Theorem.
Let us show that φ is surjective. Let us write r for rotation 60 degrees clockwise, and s for
reflection across the diagonal 1. (Please note: we shall follow the conventions of the textbook
that products like sr are interpreted as “first do r, then do s.”) Then φ(r) = (132) and
φ(s) = (23).
Now im(φ), being a subgroup of S3 , necessarily contains h(132)i = {1, (132), (123)}. But
it also contains φ(s) = (23). Therefore | im(φ)| ≥ 4, but | im(φ)| divides 6 by Lagrange’s
theorem; therefore im(φ) = S3 .
Finally we claim ker(φ) = N . For the inclusion ⊇, note that 1 and r3 do indeed send each
diagonal to itself (r3 does so by flipping each diagonal). On the other hand, since rigid
motions send adjacent vertices to adjacent vertices, any rigid motion in ker(φ) that flips one
diagonal flips them all, and any rigid motion in ker(φ) that fixes one diagonal fixes them all.
This shows the claim.

9
Problem Set 8

1. In class today (March 21), you formed groups of five students each with four students remain-
ing, and then groups of seven students each with three students remaining. In your opinion,
how many students came to class today?

Solution. Since the natural map Z/35Z → Z/5Z × Z/7Z discussed in class sends 24 to (4, 3),
we conclude from the Chinese Remainder Theorem that the number of students present must
have been 24 (mod 35). I think there were 24 + 35 = 59 students present.

2. Let m, n ≥ 1 be integers. Prove that the set of numbers appearing as the order of some
element of Z/mZ × Z/nZ is

{d ∈ Z>0 : d divides lcm(m, n)}.

Solution. Let us first establish a lemma: for any group G, if g ∈ G has order n, and n = cd
for integers c, d ≥ 1, then g d has order c. Indeed, (g d )c = 1 but for any positive number
0 0
c0 < c, note dc0 < dc = |g|, so (g d )c = g dc 6= 1.
Let’s write ` = lcm(m, n) for short. We claim that |(1, 1)| = `. Indeed, note that (`, `) = (0, 0)
since m|` and n|`. On the other hand, if (d, d) = (0, 0) for d ∈ Z, then m|d and n|d, hence
`|d (as established in class). Hence |(1, 1)| = `, and by the lemma, every positive divisor of `
occurs as the order of some element of the form (d, d).
Conversely, if (a, b) has order d, then d necessarily divides ` since (a`, b`) = (0, 0).

3. Let G be a group and let


∆ = {(g, g) : g ∈ G} ⊆ G × G.
Prove that ∆ ≤ G × G and that ∆ is normal if and only if G is abelian.

Solution. ∆ is often called the diagonal. I leave it to you to check ∆ is a subgroup. I also
leave it to you to check that G × G is abelian if and only if G is abelian. Thus the “if”
direction follows since every subgroup of an abelian group is normal. Conversely, suppose ∆
is normal, and given g, h ∈ G, let us prove gh = hg, or equivalently g = hgh−1 . We have

∆ 3 (h, 1)(g, g)(h, 1)−1 = (hgh−1 , g),

implying that hgh−1 = g as desired.

10
4. Let H, K be groups and let φ : K → Aut(H) be a homomorphism. Recall that G = H oφ K
contains subgroups
e = {(h, 1) : h ∈ H} E G
H e = {(1, k) : k ∈ K} ≤ G
K
isomorphic to H and K respectively. Prove that K
e E G if and only if φ is trivial.
(The map φ being trivial means that φ(k) = idH for all k ∈ K.)

Solution. If φ is trivial, then k · h = h for all h ∈ H, k ∈ K (here · denotes the action of k


on h via the map φ. Then for all h ∈ H, k, k 0 ∈ K, we have
(h, k)(1, k 0 )(h, k)−1 = (h, kk 0 )(h−1 , k −1 ) = (hh−1 , kk 0 k −1 ) = (1, kk 0 k −1 ) ∈ K,
e

so K
e E G.
e E G. Given h ∈ H, k ∈ K, we wish to show k · h = h. Note
Conversely, assume K
(h−1 , 1)(1, k)(h−1 , 1)−1 = (h−1 , k)(h, 1) = (h−1 (k · h), k).
e by assumption, so h−1 (k · h) = 1, so k · h = h as desired.
This last element is in K

5. Consider the group of rigid motions of R2 , i.e. rotations, reflections, translations, and compo-
sitions of these. (Optional, ungraded: express this group as a nontrivial semidirect product.)
For each of the following subsets S of R2 , let G be the group of rigid motions of R2 that
preserves S. Exhibit G as a nontrivial semidirect product in each case. Brief explanations
may be helpful, but full proofs are not necessary.

(a) S = {(x, y) ∈ R2 : x2 + y 2 = 1}, the unit circle

Solution.
R/2πZ o Z/2Z where the nontrivial element of Z/2Z acts on the group of rotations
R/2πZ by negation. (Same as the cylinder example in class.)

(b) S = Z2 , the integer lattice points

Solution.
One possible solution: Z2 o D8 , where D8 is identified with the group of rotations and
reflections preserving the square {(x, y) ∈ R2 : −1 ≤ x, y ≤ 1} (these are precisely the
rigid motions of the plane that fix the origin and preserve Z2 ), and acts on Z2 according
to the natural action of this group on the plane R2 .

11
(c) S = {(x, y) ∈ R2 : −1 ≤ y ≤ 1}, an infinite horizontal strip.

Solution. Z/2Z o (R × Z/2Z), where the second factor records translations and reflec-
tion across the x-axis, and the first factor records reflection across the y-axis. Here, the
action Z/2Z → Aut(R × Z/2Z) sends the nonidentity element of Z/2Z to the automor-
phism of R × Z/2Z acting by negation on R and trivially on Z/2Z.
(There are other equally good ways to describe the same group, such as (Z/2Z×Z/2Z)oφ
R, with φ defined appropriately.)

6. Let p and q be distinct primes. Prove that any group of order pq is a semidirect product of
two of its proper subgroups.

Solution. By Sylow’s theorem such a group G has subgroups K and H of order p and q
respectively. Assume without loss of generality that p < q; then nq ≡ 1 mod q and nq |p
implies nq = 1, hence H is normal.
Moreover H ∩ K = 1, since (|H|, |K|) = 1, and the fact that HK/H ∼= K/(K ∩ H) (Second
Isomorphism Theorem) implies |HK| = |H||K| so HK = G.
As shown in class, these hypotheses imply that G ∼
= H o K.

12
Problem Set 9

4. Let I be the ideal of Z[x] consisting of polynomials whose constant term, coefficient of x, and
coefficient of x2 are zero. Identify, with proof, the nilradical of Z[x]/I.

Solution. Write f = f + I for short. We claim the nilradical is (x), that is, the nilradical is
comprised of the classes of polynomials with zero constant term. If f has zero constant term,
3
then f 3 ∈ I, so f = 0 in Z[x]/I. Conversely, if f has constant term a0 6= 0, then f n has
n
constant term an0 6= 0, so f 6= 0 ∈ Z[x]/I for any n ≥ 0.

5. Let I be an ideal in a ring R, and let π : R → R/I be the natural projection. Prove the
following universal property of quotients: If ϕ : R → S is any ring homomorphism such that
I ⊆ ker(φ), then there exists a unique homomorphism ϕ : R/I → S such that

ϕ = ϕ ◦ π.

Solution. If such a ϕ exists, then given any a ∈ R, the equation above implies

ϕ(π(a)) = ϕ(a + I) = ϕ(a).

Thus ϕ is uniquely determined, if it exists.


Now we check that ϕ(a + I) = ϕ(a) is a well-defined homomorphism. Given a, a0 ∈ R, if
a + I = a0 + I then a − a0 ∈ I ⊆ ker(ϕ), hence ϕ(a) = ϕ(a0 ). This shows well-definedness. I
leave it to you to check that ϕ is a homomorphism of rings.

13
Problem Set 10

1. Let R be an integral domain, and let f, g ∈ R. Prove that (f ) = (g) if and only if f = ga for
some unit a.

Solution. Let us prove the ⇒ direction; I leave the other direction to you. If (f ) = (g) then
f = ga and g = f b for some a, b ∈ R. Thus f = abf and f (1 − ab) = 0. Now if f = 0 then
necessarily g = 0 and we are done; if f 6= 0 then 1 − ab = 0 since R has no zero divisors. So
a and b are units.

2. Let K be a field, and consider the ring K[[x]] of formal power series.

(a) Prove that K[[x]] is an integral domain.

Solution. Suppose f, g ∈ K[[x]] are nonzero; we wish to show f g is nonzero. Define


the order of a nonzero power series f to be the smallest exponent appearing in f .
Let d and e denote the orders of f and g respectively, and let ad and be be the coefficient
of xd in f and the coefficient of xe in g, respectively. Note ad , be 6= 0.
Then the coefficient of xd+e in f g is ad be , which is nonzero since K is a field and hence
has no zerodivisors.
Thus f g 6= 0, since we have exhibited a nonzero coefficient in it.

(b) Prove that the ideals of K[[x]] are 0 or are of the form (xn ) for some integer n ≥ 0.

Solution. Let I ⊆ K[[x]] be an ideal. If I = 0 we are done. Otherwise, let f ∈ I be a


nonzero element of smallest order d.
We claim I = (xd ). The inclusion I ⊆ (xd ) follows from the fact that every element of
I has order at least d and hence is divisible by xd .
The inclusion I ⊇ (xd ) follows from the fact that we may factor f = xd · g for g ∈ K[[x]]
a power series with nonzero constant term. Such power series are units, by a previous
homework problem. Therefore xd ∈ I, as desired.

(c) Which of the ideals of K[[x]] are principal? maximal? prime? Prove your answers.

Solution. Each ideal is generated by 1 element (including the 0 ideal which is generated
by 0), so they are are all principal.
Note (x) ⊃ (x2 ) ⊃ · · · ⊃ 0 and (x) is thus the unique maximal ideal.

14
Note that (x) is therefore prime, being maximal; and 0 is prime since K[[x]] is an integral
domain. On the other hand, we claim that for d ≥ 2 the ideal I = (xd ) is not prime,
since x · xd−1 ∈ I but x 6∈ I and xd−1 6∈ I, which I leave to you to verify.

5. This is a problem in beginning algebraic geometry. Given an ideal I ⊆ R[x, y], we let the
vanishing locus or variety of I be the subset of R2

V (I) = {(a, b) ∈ R2 : f (a, b) = 0 for all f ∈ I}.

(a) Prove that if I = (f1 , . . . , fn ) then V (I) = {(a, b) ∈ R2 : fi (a, b) = 0 for all i = 1, . . . , n}.
(b) Draw pictures of V (I) for I = (y(y − x2 )) and for I = (x − y, y − x3 ).
(c) Prove that if I1 ⊆ I2 are ideals of R[x, y] then V (I1 ) ⊇ V (I2 ).
(d) Using part (c) to help, prove the identities

V (I + J) = V (I) ∩ V (J) and V (IJ) = V (I ∩ J) = V (I) ∪ V (J).

Check your results in part (b) accordingly.

Solution. I will prove the second statement of (d) only and leave the rest to you. This is
the statement that
V (IJ) = V (I ∩ J) = V (I) ∪ V (J).
To prove it, let us begin by noting

IJ ⊆ I ∩ J ⊆ I and IJ ⊆ I ∩ J ⊆ J.

So by part (c), we have


V (IJ) ⊇ V (I ∩ J) ⊇ V (I) ∪ V (J).
It remains only to prove
V (IJ) ⊆ V (I) ∪ V (J).

Suppose there is a point (a, b) ∈ R2 in neither V (I) nor V (J). This means there exists f ∈ I
and g ∈ J with f (a, b) 6= 0 and g(a, b) 6= 0. Therefore (f g)(a, b) 6= 0. Since f g ∈ IJ, we
conclude (a, b) 6∈ V (IJ), as desired.

15
Problem Set 11

2. Let K be a field. Prove that K[[x]] is a Euclidean domain with respect to the following norm:
N (0) = 0, and for all nonzero p ∈ K[[x]], N (p) is the order of p, i.e. the smallest exponent
appearing in p.

Solution. First, we claim the following, using what you know about K[[x]] from last week’s
homework: if f, g ∈ K[[x]] are nonzero power series, then g|f if and only if ord(f ) ≥ ord(g).
I leave the proof of the above claim to you.
Now, suppose f, g ∈ K[[x]] and g 6= 0. If f = 0 then
f = 0 · g + 0.
If f 6= 0 and ord(f ) < ord(g), then
f = 0 · g + f.
Finally, if f 6= 0 and ord(f ) ≥ ord(g), then
f =h·g+0
for some h, by the claim above. In each case, the Euclidean property is satisfied.
(By the way, this problem is an instance of the more general statement that discrete valuation
rings are Euclidean domains).

4. Compute a gcd of 4 + 2i and 5i in Z[i]. Identifying Z[i] with the integer lattice points in the
complex plane, draw a picture of the elements of the ideal (4 + 2i, 5i).

Solution. We have
5i = i(4 + 2i) + (2 + i)
4 + 2i = 2(2 + i) + 0
So 2 + i is a gcd, and the ideal (4 + 2i, 5i) = (2 + i) looks like a square lattice, generated
additively by 2 + i and −1 + 2i.

5. Let R be an integral domain. We defined the field of fractions K, whose elements are equiv-
alence classes of {(a, b) : a, b ∈ R, b 6= 0} where (a, b) ∼ (c, d) if ad = bc. We write a/b for the
class of (a, b). We defined
a/b + c/d = (ad + bc)/bd, a/b · c/d = (ac)/(bd).
Convince yourself that + and · are well-defined and make K into a field with 0 = 0/1 and
1 = 1/1 (ungraded).

16
(a) Prove that the map i : R → K given by i(r) = r/1 is a ring homomorphism sending all
nonzero elements to units.

Solution. Given r, r0 ∈ R, we check

i(r + r0 ) = (r + r0 )/1 = r/1 + r0 /1 = i(r) + i(r0 )

and
i(rr0 ) = (rr0 )/1 = i(r)i(r0 ).
Moreover if r 6= 0 then r/1 is a unit since r/1 · 1/r = 1.

(b) Prove the following universal property of localization: Let S be a commutative ring with
1. If f : R → S is any ring homomorphism sending all nonzero elements of R to units of
S, then there is a unique ring homomorphism fe: K → S such that

f = fe ◦ i.

Solution. First we establish that f (1) = 1. We have

f (1) = f (1 · 1) = f (1) · f (1).

But f (1) is a unit in S by assumption, so we conclude 1 = f (1).


Next, if fe: K → S is a ring homomorphism satisfying the conditions above, then
fe(r/1) = f (r) for all r 6= 0. This implies

fe(r/1) · fe(1/r) = fe(1) = f (1) = 1,

and since fe(r/1) = f (r) is assumed to be a unit, the equation above implies fe(1/r) =
f (r)−1 . Therefore, for all r, s ∈ R with r 6= 0,

fe(s/r) = fe(s/1 · 1/r) = f (s)f (r)−1 .

This shows uniqueness of fe, if it exists.


Finally, we check existence, i.e. that fe(s/r) := f (s)f (r)−1 really is a well-defined ring
homomorphism with f = fe ◦ i. Suppose s0 /r0 = s/r, so s0 r = sr0 . Then f (s0 )f (r) =
f (s)f (r0 ), and
f (s0 )f (r0 )−1 = f (s)f (r)−1 ,
which shows that fe is well-defined. I leave it to you to check that fe is a ring homomor-
phism with f = fe ◦ i.

17
Problem Set 12

1. Let R = {(a1 , a2 , a3 , . . .) : ai ∈ Z}, i.e., R is the ring of infinite tuples of Z, indexed by the
positive integers, with coordinatewise addition and multiplication. For each j = 1, 2, . . . let

Ij = {(a1 , a2 , a3 . . .) ∈ R : ai = 0 for all i ≥ j.}

(a) Show that the Ij are principal idealsSforming an ascending chain I1 ( I2 ( · · · that
doesn’t stabilize. Conclude that I = j≥1 Ij an ideal that is not finitely generated.

Solution. Write ej for the element which is j ones followed by all zeroes. Note that
Ij = {(a1 , a2 , . . .) · ej : ai ∈ Z} = (ej ), so in particular Ij is a principal ideal.
For each j, we have Ij ( Ij+1 directly from the definitions; in particular ej+1 ∈ Ij+1 \ Ij
shows that Ij is properly contained in Ij+1 .
Therefore I is not finitely generated, since if instead I = (f1 , . . . , fn ), then there are
indices j1 , . . . , jn such that fi ∈ Iji . Let j = max(j1 , . . . , jn ). Then f1 , . . . , fn ∈ Ij . But
this would imply that Ij = I, contradicting that the ideals Ij+1 , Ij+2 , . . . are strictly
bigger than Ij .

(b) Is I prime?

Solution. No. Note that I consists of those sequences of integers which are eventually
zero. Then writing a = (1, 0, 1, 0, . . .) and b = (0, 1, 0, 1, . . .), we have ab = 0 ∈ I but
a, b 6∈ I.

3. Convince yourself that the polynomial x3 + x + 1 is irreducible in F2 [x]. Write out the
multiplication table for the 8-element field K = F2 [x]/(x3 + x + 1), and check that the
multiplicative group of nonzero elements in K is isomorphic to Z/7Z.

Solution. If x3 + x + 1 were reducible, then one of its factors would have to have degree 1,
since it has degree 3, i.e. it would have a solution over F2 . But it doesn’t (as seen by plugging
in 0 and 1.)
The elements of K are (the classes of)

0, 1, x, x + 1, x2 , x2 + 1, x2 + x, x2 + x + 1.

I leave the full multiplication table to you, but here is one example. Let us compute x2 ·(x2 +1)
in K. We use the division algorithm, dividing x4 + x2 by x3 + x + 1 to get

x4 + x2 = x(x3 + x + 1) + x

18
So in K, we have x2 · (x2 + 1) = x.

4. For K a field of characteristic p > 0, we define the Frobenius map e : K → K by e(a) = ap .


Show that e is a homomorphism. (To show that e(a + b) = e(a) + e(b) you may wish to appeal
to the Binomial Theorem.)
Also, compute what e does to each element of the field K from Problem 3.

Solution. We have e(ab) = (ab)p = ap bp = e(a)e(b) by commutativity of multiplication.


Next, we have
(a + b)p = ap + (stuff) + bp ,
where the stuff in the middle consists of terms of the form pi · ai bp−i where


 
p p!
= .
i i!(p − i)!

by pi we officially mean 1 + · · · + 1 ∈ K where there are pi summands. The


 
(To be clear,
integer pi is a multiple of p, since the numerator has a factor of p but not the denominator.


Since k has characteristic p, we conclude that (a + b)p = ap + bp .


As for the specific field K from problem 3, I computed that e sends the elements

0, 1, x, x + 1, x2 , x2 + 1, x2 + x, x2 + x + 1

to
0, 1, x2 , x2 + 1, x2 + x, x2 + x + 1, x, x + 1
respectively.
(In particular, you may check that e is actually an automorphism of K whenever K is a finite
field.)

19

Você também pode gostar